Problem of primes

A prime q q divides both n 2 + 3 n^2 + 3 and n 2 + 2 n + 4 n^2 +2n +4 for some integer n n .

The remainder when n n is divided by q q is r r . Find q + r q + r .

19 15 11 13

This section requires Javascript.
You are seeing this because something didn't load right. We suggest you, (a) try refreshing the page, (b) enabling javascript if it is disabled on your browser and, finally, (c) loading the non-javascript version of this page . We're sorry about the hassle.

1 solution

Arpan Mathur
Feb 8, 2017

Since q n 2 + 3 q | n^2 + 3 and q n 2 + 2 n + 4 q 2 n + 1 2 n + 1 = k q . . . ( 1 ) q | n^2 + 2n + 4 \Rightarrow q | 2n + 1 \Rightarrow 2n + 1 = k q ...(1) , where k is integral.

Also q n 2 + 3 n 2 + 3 = m q . . . ( 2 ) q | n^2 + 3 \Rightarrow n^2 + 3 = mq ...(2) , where m is integral.

Eliminating n n in Equations ( 1 ) (1) and ( 2 ) (2) gives 4 m q + 2 k q k 2 q 2 = 13 q ( 4 m + 2 k k 2 q ) = 13 q 13 q = 13 4mq + 2kq -k^2q^2= 13 \Rightarrow q(4m + 2k -k^2q) = 13 \Rightarrow q |13 \Rightarrow \boxed{q = 13} .

Now ( 1 ) (1) and ( 2 ) (2) may be rewritten as congruences:

2 n 1 ( m o d 13 ) 2n \equiv -1 \pmod{13} and n 2 3 ( m o d 13 ) n^2 \equiv -3 \pmod{13} . Solving both simultaneously yields n 6 ( m o d 13 ) n \equiv 6 \pmod{13} i.e r = 6 \boxed{r = 6} .

Thus q + r = 19 \boxed{q + r = 19} .

To elaborate on what you're doing in the first half of the solution (and disspelling some of the magic that seems to be happening), essentially you are looking for polynomial combinations of n 2 + 2 n + 4 , n 2 + 3 n^2 + 2n + 4, n^2 + 3 that will give a linear term. In such a case, using the euclidean algorithm gives us a deterministic process, as opposed to having to "guess" at how to eliminate n n . We have

( n 2 + 2 n + 4 ) ( n 2 + 3 ) = 2 n + 1 4 ( n 2 + 3 ) ( 2 n 1 ) ( 2 n + 1 ) = 13 \begin{array} { l l l l l l } (n^2 + 2n + 4) & - ( n^2 + 3) & = & 2n + 1 \\ 4(n^2 + 3) & - (2n-1)(2n+1) & = & 13 \\ \end{array}

Hence, ( 2 n + 1 ) ( n 2 + 2 n + 4 ) + ( 2 n + 3 ) ( n 2 + 3 ) = 13 ( - 2n+1) ( n^2 + 2n + 4 ) + (2n+3) ( n^2 + 3) = 13 .

Note: The identity q ( 4 m + 2 k k 2 q ) = 13 q ( 4m + 2k - k^2 q) = 13 in the solution translates into
4 ( n 2 + 3 ) + 2 ( ( n 2 + 2 n + 4 ) ( n 2 + 3 ) ) ( ( n 2 + 2 n + 4 ) ( n 2 + 3 ) ) 2 = 13 4 (n^2 + 3 ) + 2 \left( (n^2 + 2n + 4) - ( n^2 + 3) \right) - \left( (n^2 + 2n + 4) - ( n^2 + 3) \right) ^2 = 13 ,
When carefully expanded, this does indeed lead to the equation ( 2 n + 1 ) ( n 2 + 2 n + 4 ) + ( 2 n + 3 ) ( n 2 + 3 ) = 13 ( - 2n+1) ( n^2 + 2n + 4 ) + (2n+3) ( n^2 + 3) = 13 .

Calvin Lin Staff - 4 years, 4 months ago

@Calvin Lin Thanks

Arpan Mathur - 4 years, 4 months ago

0 pending reports

×

Problem Loading...

Note Loading...

Set Loading...